Đến nội dung

Cetus nội dung

Có 41 mục bởi Cetus (Tìm giới hạn từ 26-04-2020)



Sắp theo                Sắp xếp  

#608319 $\left\{\begin{matrix}x^4-4x^2+y^2-6y+9=0...

Đã gửi bởi Cetus on 10-01-2016 - 17:46 trong Phương trình - hệ phương trình - bất phương trình

:B): đương nhiên :wacko:

 Lần sau mong bạn làm ra cụ thể hơn, không nên chỉ diễn giải bằng lời như vậy  :ukliam2:  :ukliam2:  :ukliam2:




#590427 chọn đội tuyển trường chuyên Nguyễn Du-Đaklak 2015-2016(vòng 1)

Đã gửi bởi Cetus on 23-09-2015 - 12:19 trong Thi HSG cấp Tỉnh, Thành phố. Olympic 30-4. Đề thi và kiểm tra đội tuyển các cấp.

BĐT lấy của THTT ak??? :ukliam2:  :ukliam2:  :ukliam2:




#575788 Tìm GTLN của $P=(\left | x-y \right |+1)(\left | y-z...

Đã gửi bởi Cetus on 27-07-2015 - 10:11 trong Bất đẳng thức - Cực trị

Cho các số thực không âm x,y,z thỏa mãn $max\left \{ \left | x-y \right |,\left | y-z \right |,\left | z-x \right | \right \}\leq 2$

và $xy+yz+zx=2$

Tìm GTLN của $P=(\left | x-y \right |+1)(\left | y-z \right |+1)(\left | z-x \right |+1)-\sqrt{x^{2}+y^{2}+z^{2}}$




#575785 Tìm GTNN của $P=x^{4}+y^{4}+z^{4}$

Đã gửi bởi Cetus on 27-07-2015 - 10:06 trong Bất đẳng thức - Cực trị

Cho x,y,z là các số dương thỏa mãn: $(x-y)^{2}+(y-z)^{2}+(z-x)^{2}=8$ và $x^{3}+y^{3}+z^{3}=1$

Tìm GTNN của $P=x^{4}+y^{4}+z^{4}$




#569725 Bánh canh chém gió về kì thi IMO 2015

Đã gửi bởi Cetus on 03-07-2015 - 21:16 trong Thi HSG Quốc gia và Quốc tế

VMF mà tham gia IMO thì hính như chỉ có Phạm Kim Hùng thôi thì phải. Còn mà việc học sinh không phải trường chuyên mà được đi thi IMO thì theo em là rất khó :D. Nhiều người tài năng của các trường chuyên trên cả nước thậm chí còn không được đi thi QG nữa là học sinh không phải trường chuyên học khác chương trình. Bản thân e không học trường chuyên nên chuyện này e hiểu cũng khá rõ :D




#565731 Giải bất phương trình sau $1+\sqrt{x-1}\left (...

Đã gửi bởi Cetus on 14-06-2015 - 20:05 trong Phương trình - hệ phương trình - bất phương trình

Bài toán. (Đề thi thử đại học môn toán chuyên Vĩnh Phúc lần 4 năm 2015) Giải bất phương trình sau $1+\sqrt{x-1}\left ( \sqrt{2x}-3\sqrt{x-1} \right )^{3}\geq 0$.

 Hình như là chuyên Đại học Vinh bác ạ :D

 

File gửi kèm




#565120 $\left\{\begin{matrix} (x+1+\sqrt...

Đã gửi bởi Cetus on 12-06-2015 - 10:51 trong Phương trình - hệ phương trình - bất phương trình

1) $\left\{\begin{matrix} (x+1+\sqrt{x^{2}+3x+2})(\sqrt{y^{2}+1}-y)=y\\ \sqrt{2x^{2}-16y^{2}+42}=1+\sqrt{2x-3} \end{matrix}\right.$

2) $\left\{\begin{matrix} (x^{2}+4)(x-2)(\sqrt{y-1}+2)=y^{2}-2y+15\\ y-10x+11+x\sqrt{5x-6}=0 \end{matrix}\right.$

3) $\left\{\begin{matrix} (x+2)\sqrt{x}+3x=y^{3}-y+1\\ 7\sqrt{x-1}+14\sqrt{y^{2}-2y-3}-8=5y^{2}-10y+4\sqrt{x^{2}-5x+4} \end{matrix}\right.$

Câu cuối e kt lại đề cái anh mới lm đc 1 câu thôi:

Câu 1: Liên hợp rồi đưa pt đầu về dạng: $x+1+\sqrt{x^{2}+3x+2}=y\sqrt{y^{2}+1}+y^{2}=\sqrt{y^{4}+y^{2}}+y^{2}=(x+1)+\sqrt{(x+1)^{2}+x+1}\rightarrow y^{2}=x+1$




#561966 $(3x-4)\sqrt{3x-2}-4x^{3}+9x^{2}-7x=0...

Đã gửi bởi Cetus on 27-05-2015 - 20:38 trong Phương trình - hệ phương trình - bất phương trình

Check lại đề xem, mình solve ra vô nghiệm  :(

Đề đúng mà bạn




#561872 $(3x-4)\sqrt{3x-2}-4x^{3}+9x^{2}-7x=0...

Đã gửi bởi Cetus on 27-05-2015 - 12:46 trong Phương trình - hệ phương trình - bất phương trình

Giải pt:

$(3x-4)\sqrt{3x-2}-4x^{3}+9x^{2}-7x=0$




#561871 $(2x-11)(\sqrt{3x-8}-\sqrt{x+1})=5$

Đã gửi bởi Cetus on 27-05-2015 - 12:44 trong Phương trình - hệ phương trình - bất phương trình

Giải Pt sau:

$(2x-11)(\sqrt{3x-8}-\sqrt{x+1})=5$




#561384 $(x+5)\sqrt{x+1}+1=\sqrt[3]{3x+4}$

Đã gửi bởi Cetus on 24-05-2015 - 20:15 trong Phương trình - hệ phương trình - bất phương trình

Giải phương trình: $(x+5)\sqrt{x+1}+1=\sqrt[3]{3x+4}$

(Giải bằng cách lập hệ)

 

Đặt: $\sqrt{x+1}=a, \sqrt[3]{3x+4}=b$

Ta có hệ pt: $\left\{\begin{matrix} a^{3}+4a=b-1 & \\ b^{3}-1=3a^{2} & \end{matrix}\right.$

Cộng hai pt trên , ta đc: $a^{3}+3a^{2}+3a+1+a+1=b^{3}+b\Leftrightarrow (a+1)^{3}+(a+1)=b^{3}+b$

...




#558671 $\sum x^{2}\geq \sum x$

Đã gửi bởi Cetus on 10-05-2015 - 21:08 trong Bất đẳng thức và cực trị

3. Cho $a,b,c>0$. CMR:

$\sum \frac{2\sqrt{x}}{x^{3}+y^{2}}\leq \sum \frac{1}{x^{2}}$

 

 

Bạn có thể ghi rõ cho mình điều kiện của bài được không, nhìn khó hiểu quá :D




#558667 $\boxed{TOPIC}$ Các đề thi ôn luyện tuyển sinh vào t...

Đã gửi bởi Cetus on 10-05-2015 - 21:03 trong Tài liệu - Đề thi

Cho mình đăng kí tham gia được không bạn???  :icon6:  :icon6:  :icon6:




#554414 Giải hệ phương trình

Đã gửi bởi Cetus on 16-04-2015 - 17:10 trong Phương trình - hệ phương trình - bất phương trình

Giải hệ phương trình: $\left\{\begin{matrix} \frac{2}{(\sqrt{x}+\sqrt{y})^{2}}+\frac{1}{x+\sqrt{y(2x-y)}}=\frac{2}{y+\sqrt{x(2-y)}} & \\ 2(y-4)\sqrt{2x-y-3}-(x-6)\sqrt{x+y+1}=3(y-2) & \end{matrix}\right.$




#553950 $\sum \frac{a}{a+b}\geq \frac...

Đã gửi bởi Cetus on 14-04-2015 - 17:19 trong Bất đẳng thức và cực trị

Chắc là hệ quả của bài trong báo THTT  :icon6:  :icon6:  :icon6:  :icon6:  :icon6:




#552935 Giải phương trình: $(4x^{3}-x+3)^{3}-x^{3}...

Đã gửi bởi Cetus on 10-04-2015 - 15:59 trong Phương trình - hệ phương trình - bất phương trình

Giải phương trình: $(4x^{3}-x+3)^{3}-x^{3}=\frac{3}{2}$

 Học hành thế này đây.

Biến đổi pt về: $(8x^{3}-2x+6)^{3}-8x^{3}=12$

Đặt a=2x, ta được: $(a^{3}-a+6)^{3}=a^{3}+12$

Đặt tiếp: $a^{3}-a+6=t$

Ta được hệ pt: $\left\{\begin{matrix} t^{3}-a^{3}=12 & \\ t=a^{3}-a+6 & \end{matrix}\right.$

Thế các số vào vế, ta được pt: $t^{3}-a^{3}=2(t+a-a^{3})$




#550440 Đề thi học sinh giỏi trường QL I năm học 2014-2015

Đã gửi bởi Cetus on 31-03-2015 - 12:21 trong Tài liệu tham khảo khác

Quỳnh Lưu I chả bao giờ lại ra đề HSG kiểu như thế này cả  >:)  >:)  >:)  >:)  >:)




#544943 Giải hệ phương trình

Đã gửi bởi Cetus on 20-02-2015 - 08:42 trong Phương trình - hệ phương trình - bất phương trình

Giải hệ phương trình: $\left\{\begin{matrix} \sqrt{y+2x-1}+\sqrt{1-y}=y+2 & \\ x\sqrt{x}=\sqrt{y(x-1)}+\sqrt{x^{2}-y} & \end{matrix}\right.$




#544817 Chứng minh $\frac{x_{2014}+1}{2014}...

Đã gửi bởi Cetus on 18-02-2015 - 16:54 trong Dãy số - Giới hạn

Cho dãy số $x_{n}$ được xác định như sau: $x_{1}=1,x_{2}=2013,x_{n+2}=4026x_{n+1}-x_{n}$ n=1,2...

Chứng minh $\frac{x_{2014}+1}{2014}$ là số chính phương




#544815 Chứng minh rằng: $x_{2014}\vdots x_{19}$

Đã gửi bởi Cetus on 18-02-2015 - 16:48 trong Dãy số - Giới hạn

Cho dãy số $x_{n}$ xác định bởi $x_{0}=0, x_{1}=3, x_{n+1}=\frac{7x_{n}+3\sqrt{4+5x^{2}_{n}}}{2}$ với mọi số nguyên không âm n. Chứng minh rằng: $x_{2014}\vdots x_{19}$




#544313 Tìm hệ số k lớn nhất

Đã gửi bởi Cetus on 15-02-2015 - 18:16 trong Bất đẳng thức - Cực trị

Cho a,b,c là các số thực dương thỏa mãn abc=1, tìm hệ số k lớn nhất thỏa mãn BĐT:

$\frac{1}{a}+\frac{1}{b}+\frac{1}{c}+\frac{k}{a+b+c+1}\geq 3+\frac{k}{4}$




#543287 Tìm GTLN của: A=$2(x^{3}+y^{3}+z^{3})-(x^...

Đã gửi bởi Cetus on 07-02-2015 - 12:26 trong Bất đẳng thức - Cực trị

Cho các số thực x,y,z thuộc khoảng [0;1]. Tìm GTLN của: A=$2(x^{3}+y^{3}+z^{3})-(x^{2}y+y^{2}z+z^{2}x)$




#531943 Tìm GTLN của: $x^{2}y+y^{2}z+z^{2}x$

Đã gửi bởi Cetus on 05-11-2014 - 11:58 trong Bất đẳng thức - Cực trị

Cái chỗ này đề bài cho $x,y,z>0$ mà cậu, sao lại có BĐT này vậy

 

Cậu tham khảo ở đây: http://diendantoanho...le-frac427abc3/




#531776 Tìm GTLN của: $x^{2}y+y^{2}z+z^{2}x$

Đã gửi bởi Cetus on 04-11-2014 - 17:01 trong Bất đẳng thức - Cực trị

Cho x,y,z là các số thực dương thỏa mãn: x+y+z=1

Tìm GTLN của: $x^{2}y+y^{2}z+z^{2}x$




#531487 $\sqrt[3]{a}+\sqrt[3]{b}+\sqrt[3]...

Đã gửi bởi Cetus on 02-11-2014 - 10:27 trong Bất đẳng thức - Cực trị

Cho a,b,c là các số thực dương thỏa mãn:

$\frac{1}{a+1}+\frac{1}{b+1}+\frac{1}{c+1}=1$

CMR: $\sqrt[3]{a}+\sqrt[3]{b}+\sqrt[3]{c}\leq \frac{3}{2}\sqrt[3]{2abc}$